You are on page 1of 75

PROBLEMS

PROBLEM I: TRUE OR FALSE


1. All events and transactions of an entity are recognized the books
of accounts.

2. The accounting process of assigning numbers, commonly in


monetary terms, to the economic transactions and events is
referred to as classifying.
3. The basic purpose of accounting is to provide information about
economic activities intended to be useful in making economic
decisions.
4. Financial accounting is the branch of accounting that focuses on
general purpose reports of financial position and operating
results known as the financial statements.
5. General purpose financial statements are those statements that
cater to the common and specific needs of a wide range of
external users.
6. The financial statements are the only source of information when
making economic decisions.
7. All information presented in the financial statements are sourced from the accounting records of
the entity.

8. Entity A's accounting period starts on July 1 and ends on June 30 of the following year. Entity A
uses a fiscal year period.

9. Once promulgated, accounting standards are never changed.


10. The entity's management is responsible for the selection of appropriate accounting policies, not the
accountant.

PROBLEM 2: MULTIPLE CHOICE


1. The concept of recognition is applied in which of the following
instances?
a. An entity includes the effects of an event in the financial statements through a journal entry.
b. An entity removes the effects of an event from the financial statements through a journal entry.

c. An entity discloses only an event in the notes, rather than including the effects of the
event in the monetary totals in the financial statements.
An entity records an event through a memorandum entry.

2. Which of the following events is not considered an exchange or reciprocal transfer?

a. purchase of inventory on account


b. lending money to another entity
c. payment of a loan payable
d. payment of taxes

3. Which of the following events is considered a nonreciprocal


transfer?
a. sale of an asset c. loss from a calamity
b. donation d. production of finished goods

4. To be useful, accounting information should be presented using

a. monetary amounts. c. historical costs.

b. a common denominator. d. fair values.

5. Which of the following violates the historical cost concept?


a. Recording purchases of merchandise invefitory at the purchase price.
b. Recording a building at the total construction costs.
c. Measuring inventories at net realizable value.

d. Recording an equipment acquired in an installment purchase at the cash price equivalent.

6. Entity A values its fixed assets at their historical costs and does not restate them for changes in the
purchasing power of the Philippine pesos due to inflation. Entity A is applying which of the following
accounting concepts?
a. prudence c. stable monetary unit
b. accrual basis d. time period
7. Entity A engages in importing and exporting activities. At
the end of the period, Entity A has assets and liabilities
denominated in foreign currencies. When preparing its
financial statements, Entity A translates these assets and
liabilities to pesos. Entity A is most likely to be applying
which of the following accounting concepts?
a. double entry c. stable monetary unit
b. accrual basis d. time period

8. Preparing financial statements at least annually is an


application of which of the following accounting
concepts?
a. historical cost c. stable monetary unit
b. accrual basis d. time period

9. Entity A acquires merchandise inventory. Entity A


initially records the acquisition cost of the inventory as
asset rather than an outright expense. When the inventory
is subsequently sold, Entity A recognizes the cost of the
inventory sold as expense, in the same period the sale
revenue is recognized. This is an application of which of
the following accounting concepts?
a. stable monetary unit c. matching
b. materiality d. proprietary

10. On Day 1, a customer buys goods from Entity A and


promises to pay the sale price on Day 30. Entity A
recognizes sales revenue on Day 1 rather than on Day 30.
This is an application of which of the following
accounting concepts?
a. prudence c. consistency
b. accrual basis d. materiality
PROBLEM 3: MULTIPLE CHOICE

1 All of the following are considered internal events, except


a. transfer of goods from work-in-process to finished goods
inventory
b. losses from flood, earthquake, fire and other calamities
c. transformation of biological assets from immature to
mature
d. vandalism committed by the entity's employees

2. Which of the following is considered an internal user of Entity A's financial reports?

a. Entity B, a bank, requires Entity A to submit audited


financial statements in conjunction to a loan being applied
for by Entity A.
b. Mr. I is deciding whether to invest in Entity A. Mr. I uses Entity A's financial statements in
making its investment decision.
c. Ms. S, a shareholder of Entity A, is deciding whether to hold or sell her shareholdings in
Entity A. Ms. S uses Entity A's financial statements in making its "hold or sell" decision.
d. Mr. X, a member of Entity A's board of directors, uses financial reports to make decisions
regarding the financial and operational affairs of Entity A.

3. When resolving accounting problems not specifically addressed by current standards, an entity shall be
guided by the hierarchy of financial reporting standards. The correct sequence of the hierarchy of
financial reporting standards in the Philippines is
I. PASS, PFRSs and Interpretations Il. II. Conceptual
Framework.
III. Judgement

IV. Pronouncement of other standard-setting bodies

a. I, Ill, Il and IV c. 1, IV, 11 and 111


d. 1, 11, Ill and IV
b. 1, 11, IV and III

4. The proper application of accounting principles is most


dependent upon the
a. management. C. auditor
B. accountant. d. chief executive officer.
5. Which of the following statements is correct?
a. Accounting provides quantitative information only.
b. Accounting is considered an art because it requires the use
of creative skills and judgment.
c. The only acceptable measurement basis in accounting is historical cost.
d. Qualitative information can be found only in the notes to
the financial statements.

6. Which of the following statements is correct?


a. All quantitative information are also financial in nature.
b. The accounting process of assigning peso amounts to
economic transactions and events is measuring.
c. The economic activity that involves using current inputs
to increase the stock of resources available for output is
called savings.
d. The economic activity of using the final output of the
production process is called income distribution.

7. Which of the following statements is incorrect regarding accounting concepts?

a. Under the Accrual Basis of accounting, revenues are


recognized when earned and expenses are recognized
when incurred, not when cash is received and disbursed
b. Under the Going concern concept, the business entity is
assumed to carry on its operations for an indefinite period
of time.
c. Under the Business entity/ Separate entity/ Entity/
Accounting entity Concept, the business is treated
separately from its owners.
d. Under the Time Period/ Periodicity/ Accounting Period
concept, the life of the business is divided into series of
reporting periods.
e. Under the Cost-benefit concept, the cost of processing and
communicating information should exceed the benefits
derived from it.

8. Which of the following statements is incorrect?


a. Financial reporting standards may at times be influenced by legal, political, business and social
environments.

b. General-purpose financial statements must be prepared by a


Certified Public Accountant.
c. General purpose financial statements are prepared primarily
for the use of external users.
d. The PFRSs are issued by the Financial Reporting
Standards Council.

9. Mr. John Doe, CPA, is a professor in a university where he


teaches mainly home economics, music and physical
education. Those subjects require that the teacher must be
awesome. Mr. Doe is also frequently invited as a judge in
beauty pageants and singing contests and as a referee in
mixed martial arts competitions. Mr. Doe is considered to be
practicing accountancy in which of the following sectors?
a. Academe c. Commerce and industry
b. Public accounting d. None of these

10. Changes to reporting standards are primarily made in response to


a. government regulations. C. global modernization
b. users’ needs d. all of these

PROBLEM 4: FOR CLASSROOM VISCUSSION Events


1. Entity A buys bananas and converts them into banana chips. The
conversion of bananas into banana chips is a (an)
a. non-accountable event c. non-reciprocal transfer.
b. external event. d. internal event.

Valuation by fact or opinion


2. Which of the following is considered valued by fact rather than by opinion?
a. Depreciation
c. Discount on share capital
b. Cost of goods sold d. Retained earnings

Measurement Bases
3. Which of the following is not one of the several measurement
bases used in accounting?
a. historical cost c. present value

b. fair value d. all of these are used

Accounting Concepts

4. Entity A is owned by Mr. X and Ms. Y. Which of the


following transactions does not violate the separate entity
concept and therefore is appropriately recorded in the
accounting records of Entity A?
a. Mr. X purchases groceries for his home consumption.
b. Mr. X gives Ms. Y chocolate and flowers on Valentine's Day.

c. Ms. Y provides capital to Entity A.


d. Ms. Y provides capital to Entity B, another business entity.
5. Mr. A is assessing the ability of Entity A to generate future
cash and cash equivalents. In making the assessment, Mr. A
uses not only the statement of cash flows but also the other
components of a complete set of financial statements. This is
because of which of the following concepts?
a. Going concern c. Intercalation

b. Time period d. Articulation

6. Entity A acquires a stapler. Instead of recognizing the cost of the stapler as an asset to be subsequently
depreciated, Entity A immediately charges it as expense. This is an application of which of the following
concepts?

a. Prudence c. Cost-benefit
b. Materiality d. b and c

Common branches of accounting


7. What type of users' needs is catered by general purpose financial
statements?

a. common needs c. a and b


b. specific needs d. neither a nor b

Four sectors in the practice of accountancy


8. Which of the following is not among the Four Sectors in the practice of accountancy as enumerated in R.A.
9298 also known as the "Philippine Accountancy Act of 2004"?

a. Practice in Commerce and Industry


b. Practice in the Government
c. Practice in Education/Academe
d. Practice of Private Accountancy

Accounting standards
9. The Philippine Financial Reporting Standards (PFRSs) comprise:

I. Philippine Financial Reporting Standards


II. Philippine Accounting Standards
III. Interpretations
IV. Accounting Practice Statements and Implementation
Guidance c. I and Il
a. I, II, IIII d. I and III
b. I, II, III and IV

10. Which of the following Statements is incorrect regarding the PFRSs?


a. The PFRSs based on the IFRSs.
b. The financial reporting standards used in the Philippines are the same as those used
globally.
c. The PFRSs have higher authority than the PASS and Interpretations.

d. The PFRSs are accompanied by guidance. The use of such guidance is sometimes
mandatory and sometimes optional.

PROBLEMS
PROBLEM 1: TRUE OR FALSE

1. All changes in an entity's economic resources and claims to those resources result from the entity's
financial performance.

2. The qualitative characteristics of useful information apply only to the financial information provided in
the financial statements.

3. According to IFRS Practice Statement 2 Making Materiality Judgments, cost is an important


consideration when making materiality judgments.
4. When making materiality judgments, a quantitative assessment alone is not always sufficient to
conclude that an item of information is not material.

5. Materiality judgments apply only to items that are


recognized but not to those that are unrecognized.
6. The more significant the qualitative factors are, the lower the quantitative thresholds will be. Thus, an
item with a zero amount can be material in light of qualitative thresholds.
7.When making materiality judgments, an entity should judge an item's materiality only on its own and
not in combination with other information in the complete set of financial statements.
8. The Conceptual Framework and the Standards specify a uniform
quantitative threshold for materiality.
9. To meet the objectives of general purpose financial
reporting, a Standard sometimes contains requirements that
depart from the Conceptual Framework.
10. The Conceptual Framework is concerned with the provision of financial information to both external users and
internal users.

PROBLEM 2: TRUE OR FALSE


l. The Conceptual Framework may be revised from time to time. Revisions in the Conceptual Framework
automatically result to changes in the Standards.

2. According to the revised Conceptual Framework, the asset is the right, while •the liability is the
obligation, rather than the
ultimate inflows or outflows of economic benefits resulting from the asset or liability.
3. Legal enforceability of a right, for example ownership, is
necessary for control over an economic resource to exist.
4. According to the revised Conceptual Framework, an asset can
exist even if the probability that it will provide inflows of future
economic benefits is low, and even if the asset is subject to a high
measurement uncertainty.
5. According to the revised Conceptual Framework, what the entity
controls is the right, and not the ultimate inflows of future
economic benefits that the economic resource may produce.
6. The Conceptual Framework defines income and expenses in terms of changes in assets and liabilities.
7. Not all items that meet the definition of a financial statement
element are recognized; they are recognized only if recognizing
them will also result in relevant and faithfully represented
information.
8. Measuring an asset at historical cost will always result in the same carrying amount of the asset from
period to period.

9. According to the Conceptual Framework, amortized cost


measurement relates to historical cost, rather than current value.
10. Although the use of a single measurement basis improves the understandability of the financial
statements, this may not always lead to useful information. Thus, the Standards require different
measurement bases for different assets, liabilities

income and expenses.


PROBLEM 3: MULTIPLE CHOICE
1. According to the Conceptual Framework, these are the qualitative characteristics that make information
useful to users
a. Fundamental
c. Relevance
b. Enhancing
d. Comparability
2. Information that is capable of making a difference in the

decisions made by users has this qualitative characteristic.


a. Relevance c. Timeliness

b. Faithful representation d. Verifiability

3. When making materiality judgments, the overriding


consideration is

a. the ability of the item being judged to influence users'


decisions
b. b. the size of the impact of the item being judged.
c. the characteristics of the item being judged.

d. C and d

4. This qualitative characteristic is unique in the


sense that it necessarily requires at least two items.
a. Verifiability c. Timeliness
b. Faithful representation d. Comparability

5. Which of the following enhances the comparability of


information?
a. Making unlike things look alike.
b. Making like things look different.
c. Using different methods to account for similar
transactions from period to period.
d. Consistent application of accounting policies from
period to period.

6. Information has this qualitative characteristic if different,


knowledgeable and independent observers could reach
Consensus, although not necessarily complete
agreement, that a particular depiction is a faithful
representation.
a. Relevance
b. Faithful representation
c. Verifiability
d. Comparability
7. The Conceptual Framework uses the term "claims" against the reporting entity to refer to

a. expenses. c. equity.
b. liabilities. d. both b and c

8. Entity A is assessing whether an item meets the definition of a


financial statement element. Entity A considers the transaction's
substance and economic reality rather than merely its legal form.
Entity A is applying which of the following accounting concepts?
a. Substance over form c. Accrual

b. Form over substance d. Verifiability

9. Which of the following is not one of the aspects in the revised definition of an asset?

a. Right
b. Potential to produce economic benefits
c. Probability of the expected inflows of economic benefits
from the asset
d. Control
10. The new definition of an asset (a liability) focuses on the asset (liability) being

a. a present right (obligation) that has resulted from past events and has the potential to produce
(cause a transfer of) economic benefits.

b. the expected inflows (outflows) of economic benefits that are both probable and can be
measured reliably.
c. a physical object (a duty to pay cash or other resources).
d. All of these.

11. Which of the following is not an indication of an economic resource's potential to produce economic
benefits?

a. The economic resource can be used in combination with


other resources to produce goods for sale

b. The economic resource can be used to pay liabilities.


c. The economic resource can be distributed to the owners.
d. The resource has no use in the entity's operations and has no resale value.

12. Which of the following does not meet the definition of an asset?
a. Equipment that the entity intends, and is very certain, to
acquire in the future.
b. Inventories purchased and received but not yet paid.
c. Land received from a donation.
d. A publishing title for a college textbook. The publishing title
has no physical substance, meaning you cannot see or touch
it.

13. Entity A determined that an asset exists. However, the asset's low
probability of inflows of economic benefits and its very high level
of measurement uncertainty affected Entity A's recognition
decisions about the asset, as these raised doubt on whether the
asset's recognition would result in useful information.
Consequently, Entity A did not recognize the asset, but because
Entity A deemed it relevant, information about the asset was
nonetheless provided in the notes. Which of the following
statements is correct?
a. Entity A's accounting treatment is grossly incorrect because,
according to the Conceptual Framework, all items that meet the
definition of an asset should always be recognized, regardless
of the asset's potential to produce economic benefits and its
measurement uncertainty.
b. Entity A's treatment for the asset is acceptable. The asset
is referred to as an 'unrecognized' asset.

c. Entity A's treatment for the asset is acceptable. The asset is


referred to as a 'non-existent' asset.
d' Entity A's non-recognition of the asset is correct. However,
the asset should have been completely ignored as
Providing information about unrecognized items in the
notes is not acceptable under the Conceptual Framework.
14. Which of the following will most likely affect the determination of whether an asset or a liability exists?
a. A low probability that the asset or liability will cause inflows or outflows of future economic
benefits.
b. A high level of measurement uncertainty regarding the asset or liability.

c. An unresolved dispute over a right or obligation.


d. All of these are relevant in determining the existence of an asset or a liability, according to the
Conceptual Framework.

15. An increase in the carrying amount of an asset could not possibly result in
a. the recognition of an income.
b. the recognition of an expense.
c. an increase in total equity.
d. no change in total equity.

PROBLEM 4: MULTIPLE CHOICE


1. The Conceptual Framework is least applicable in which of the following cases?
a. to account for a transaction that is specifically dealt with by a Standard
b. in resolving issues not addressed directly by a Standard

c. in developing Standards
d. in analyzing and interpreting Standards
2. General purpose financial statements are designed to
a. meet all the information needs of the primary users.

b. meet all of the common needs of all primary users.


c. meet most of the common needs of most primary users
d. meet none of the needs of users of financial information

3. These are users of financial information who are not in a position to require a reporting entity to prepare
repo tailored to their particular information needs.
a. Primary users c. Heavy users
b. Secondary users d. Slight users

4. Which of the following is not one of the primary users listed in the
Conceptual Framework?
a. Investors c. Creditors

b. Lenders d. Debtors
5. Which of the following would least likely to need general purpose financial statements in making
economic decisions?

a. Stockholders c. Management
b. Potential investors d. Lenders

6. Which of the following is not a factor to consider when applying the qualitative characteristics?

a. The information must be both relevant and faithfully represented for it to be useful.
b. The enhancing qualitative characteristics only enhance the usefulness of information but cannot make
irrelevant information or erroneous information to be useful.
c. Sometimes, it may be necessary to make trade-offs between the qualitative characteristics in order to
provide useful information.
d. To be useful, information need only to meet one, but not necessarily all, of the qualitative
characteristics.

7. Which of the following is an example of a qualitative factor used in making materiality judgments?

a. 10% of total revenues


b. 2.5% of total assets
c. 25,000 or more
d. The context of an item in relation to a current crisis in the banking and insurance industry.
8. According to the Conceptual Framework, this information
Provides a direct indication of how well management has discharged its responsibilities to make efficient

and effective use of the reporting entity's resources.

a. The changes in the reporting entity's economic resources and claims to those resources.

b. The return that the entity has produced from its economic
resources.
c. The level of the entity's economic resources in relation to the claims thereof.

d. The entity/s liquidity and solvency.

9. Which of the following statements about the concepts in the Conceptual Framework is least accurate?

a. General purpose financial reports are intended to meet equally the needs of all types of
external users.

b. A low probability of expected inflows or outflows of economic benefits resulting from an asset
or liability may affect the recognition of that asset or liability, but not necessarily its existence.

c. A high level of measurement uncertainty associated with an asset or liability can affect the
faithful representation of that asset or liability, but not necessarily its relevance.
d. Recognition means including an item in the totals of the financial statements when that item
meets the definition of a financial statement element and recognizing it would result in useful
information.

10. According to the revised Conceptual Framework, the degree Of uncertainty in the expected inflows or
outflows of economic benefits from an asset or liability or the degree Of measurement uncertainty
associated with that asset or liability
a. does not necessarily affect the conclusion that an asset or a liability exists, although it may affect
recognition decisions about the asset or liability.

b. greatly affects the conclusion that an asset or a liability


exists if the expectation of inflows or outflows is IoW
or the measurement uncertainty is high.
c. may not always affect the conclusion that an asset or a liability exists, but will most certainly result to
the nonrecognition of an asset or liability if the expectation of inflows or outflows is low or the
measurement uncertainty is high.
d. is irrelevant, both in determining the existence of an asset or a liability and in making recognition
decisions about that asset or liability.

PROBLEM 5: FOR CLASSROOM DISCUSSION


Purpose
1. Which of the following statements is incorrect regarding the purpose of the Conceptual Framework?

a. The Conceptual Framework is intended to provide a foundation


for the development of globally acceptable Standards.
b. Globally acceptable Standards contribute to economic efficiency by lowering the cost of capital and
reducing international reporting costs.
c. Globally acceptable Standards reduces the information gap between financial statement users and
the reporting entity's management.

d. The Conceptual Framework prescribes the concepts for both general purpose and specific purpose
financial reporting.

Status
2. The Conceptual Framework (choose the incorrect statement)
a. is not a PFRS.

b. in the absence of a PFRS, shall be considered by


management when making its judgment in developing and
applying an accounting policy that results in useful
information.
c. is concerned with general purpose financial reporting only.
d. prevails over the PFRSs in cases of conflicts.

Scope
3. Which of the following is excluded from the scope of th
Conceptual Framework?
a. The objective of financial reporting.
b. Qualitative characteristics of useful financial information
c. The components of a complete set of financial Statements
and their presentation requirements.
d. Definitions, recognition criteria and derecognition of
financial statement elements.
e. Descriptions of the measurement bases used in financial
reporting.

The objective of financial reporting


4. Which of the following is incorrect regarding the objective of
general purpose financial reporting?
a. The objective of general purpose financial reporting is to
provide information that is useful to primary users in
making decisions about providing resources to the entity.
b. Decisions about providing resources to the entity depend
on the users' expected returns, which in turn, depend on
assessments of the entity's prospects for future net cash
inflows and management stewardship.
c. The objective of general purpose financial reporting forms
the foundation of the Conceptual Framework.
d. General purpose financial reporting provides information
about an entity's economic resources, claims, and changes in
those resources and claims, but not on the utilization Of those
resources by the entity's management.

Primary users
5. Which of the following statements best explains why the
reporting entity's management and government regulators
are not considered primary users under the Conceptual
Framew0rk? m These users are considered related parties, and
hence do not make relevant decisions.
b. These users have the ability to curtail the operations of the reporting entity and therefore have the
ability to affect the entity's going concern.

c. These users have the power to demand information they


need directly from the reporting entity.
d. All of these.

Information on economic resources, claims, and changes


6. Information about the reporting entity's economic
resources, claims against the reporting entity and changes in
those resources and claims is referred to in the Conceptual
Framework as the
a. economic phenomena.
b. entity's return.
c. financial performance.
d. prospects for future cash flows.

Qualitative characteristics
7. Entity A deliberately overstated its liabilities from PIM to Pl.2M. What qualitative characteristic is
violated?
a. Relevance c. Timeliness

b. Faithful representation d. Understandability

8. Two primary users are using the financial information of


Entity A. If User #1 concludes that Entity A's sales has
increased while User #2 concludes that it has decreased,
Entity A's financial information is not
a. relevant. c. comparable

b. faithfully represented. d. verifiable.

Materiality
9. Entity A is making a materiality judgment. Entity A
considers the size of the impact of an item to be material if
it exceeds 5% Of total assets. What type of materiality
assessment is this?
a. Quantitative c. Requirement of a Standard
b.Qualitative d. Relevance

Financial statements and the Reporting entity


10. Which of the following is incorrect regarding the objective of general purpose financial statements?
a. General purpose financial statements show information on the reporting entity's assets, liabilities, equity,
income and
expenses.
b. General purpose financial statements are intended to show the value of the reporting entity.
c. General purpose financial statements provide information that is useful in assessing the entity's
ability to generate future net cash inflows.
d. General purpose financial statements provide information that is useful in assessing the entity's management
stewardship in relation to the use of the entity's economic resources.
The Elements of Financial Statements

11. Which of the following is least likely to be considered When


determining whether an item meets the definition of an asset?

a. whether there is a present economic resource, which is a


right, that has resulted from past events
b. whether the right has a potential to produce economic
benefits, evidenced by at least one circumstance

c. whether the entity controls the right


d. whether it is probable (more likely than not) that the
resource will produce economic benefits

12. The revised definitions of an asset and a liability emphasize


that
a. an asset is a right, and a liability is an obligation, that has
the potential to produce, or cause the transfer of, econom1C
benefits.
b. an asset is a controlled resource, and a liability is an
obligation, that is expected to cause inflows or outflows of
economic benefits.
c. an asset is the physical object and the liability is ultimate outflow of economic benefits from settling the
obligation.

d. All of these are emphasized in the revised definitions.

13. Which of the following is correct when determining the existence of an asset or a liability?
a. An asset or a liability exists if the associated right or obligation arises from legal or contractual
requirements.

b. An asset or a liability exists only if the expected inflows or outflows of economic benefits from the asset
or the liability are probable, meaning they are more likely than not to occur.

c. An asset or a liability can exist even if its potential to produce, or cause a transfer of, economic benefits
is not certain or even likely — what is important is that the right or the obligation exists in the present
and that in at least one circumstance it will produce, or cause a transfer of, economic benefits.

d. All of these are correct.


14. Control is a necessary element of an asset. Control means
a. the entity has the exclusive right over the benefits of an asset, including the ability to prevent others from
accessing those benefits.

b. that the entity can ensure that the resource will produce economic benefits in all circumstances.
c. the entity has the exclusive right over the entire economic resource, and not only a portion of it.

d. a legally enforceable right conferred to the entity by a law or other operation of law.

15. An asset is an economic resource and an economic resource is a right that has the potential to produce
economic benefits. Which of the following is not one of the potentials of an economic resource to produce
economic benefits for an entity?

a. Service potential, i.e., the resource can be used to provide


services in the entity's normal business activities.
b. The resource can be converted into cash.
c. The resource has the ability to provide cost-savings to the
entity.
d. The resource causes more outflows of cash from the
entity than inflows.
Executory contracts
16. Entity A enters into a purchase commitment with Entity B (a
seller). Neither party performs its obligation on the contract, i.e.,
Entity A did not yet pay the purchase price, while Entity B did not
yet deliver the goods. Which of the following is incorrect?
a. The contract is executory. Entity A has a combined right to
receive the goods and an obligation to pay for them.
b. Entity A recognizes neither an asset nor a liability except when
the contract becomes burdensome, such as when the goods
become obsolete before they are delivered.
c. If Entity B performs its obligation first, Entity A's combined
right and obligation changes to a liability.

d. If Entity A performs its obligation first, Entity A's combined


right and obligation changes to a liability.

Recognition and Derecognition


17. According to the revised Conceptual Framework, an item is
recognized if
a. it meets the definition of a financial statement element.
b. recognizing it would provide useful information.
c. it is probable that the item will result to an inflow or
outflow of economic benefits and its cost can be measured reliably.
d. a and b

18. According to the Conceptual Framework, an item is recognized if


it meets the definition of an asset, liability, equity, income or
expense, and recognizing it would provide relevant and
faithfully represented information. Which of the following relates to faithful representation rather than
relevance? It is uncertain whether the asset exists.

b. The asset exists but the probability that it will produce inflows of
economic benefits is low.
c. A high level of measurement uncertainty associated with the asset.

d. None of these. An item that meets the definition of an asset is always recognized as an asset.

19. Which of the following will most likely to cause the


nonrecognition of an asset or a liability?
a. The probability of an inflow (outflow) of future economic benefits from the asset (liability) is
low.
b. There is a measurement uncertainty regarding the asset or
liability.
c. It is uncertain whether the asset or liability exists.
d. Recognizing the asset or liability would not provide relevant
and faithfully represented information.

20. Which of the following would not result to the recognition of a liability?
a. Receipt of the proceeds of a bank loan.

b. Receipt of delivery of equipment purchased on credit.


c. A commitment for future execution becomes burdensome.

d. Paying in advance the purchase price of inventories for future delivery.


21. Entity A determined that a previously recognized asset no longer meet the definition of an asset.
Accordingly, Entity A removed the carrying amount of the asset from the statement of financial
position and recognized it as an expense. Entity A is applying which of the following principles?
a. Matching c. Derecognition
b. Recognition d. Presentation and disclosure
Measurement

22. Recognizing a financial statement element requires measuring it


in monetary terms. Which of the following statements incorrect
regarding measurement?
a. The Conceptual Framework only describes the measurement bases used in financial reporting
but does not specify how a particular financial statement element should be measured — this
is addressed by the Standards.
b. The Conceptual Framework broadly classifies the
measurement bases used in financial reporting into two
namely, historical cost and current value.
c. Measurement uncertainty will always cause the non recognition of a financial statement
element.
d. Measuring a financial statement element often requires
estimation.

Presentation and Disclosure

23. Effective communication makes information more useful. Effective communication requires all of
the following except
a. focusing on presentation and disclosure objectives and

principles rather than focusing on rules

b. classifying information in a manner that groups similar items and separates dissimilar items.
c. aggregating information in such a way that it is not obscured either by unnecessary detail or by
excessive aggregation.
d. using standardized descriptions, a.k.a. 'boilerplate', rather than entity-specific information.

24. According to the revised Conceptual Framework, income and


expenses are classified as either
a. recognized in profit or loss or in Other comprehensive income.
b. gains and revenues, and expenses and losses, respectively'
c. contributions from, or distributions to, the entity/s owners
d. increases or decreases in the entity's assets or
liabilities.
Concepts of Capital and Capital maintenance

25. Under this concept of capital maintenance, profit is earned if net assets increased during the
period after excluding the effects of transactions with the owners.
a. Financial capital maintenance
b. Physical capital maintenance
c. Repairs and maintenance
d. Building maintenance

PROBLEMS
PROBLEM 1: TRUE OR FALSE
1. The application of PFRSs, with additional disclosure when
necessary, is presumed to result in financial statements that
achieve a fair presentation.
2. According to PAS 1, an entity shall make an explicit and
unreserved statement of compliance with the PFRSs in the
notes only if the entity complies with all the requirements of
PFRSs.
3. PAS 1 encourages, but does not require, the presentation of
the preceding year's financial statements as comparative
information to the current year's financial statements.

4. According to PAS 1, assets and liabilities or income and


expenses are offset, unless separate presentation is required
or permitted by a PFRS,
5. According to PAS 1, PFRSs apply to financial statements as
well as to other information presented in an annual report, a
regulatory filing, or another document.
6. According to PAS 1, the line item "Cash and cash
equivalents" should always be presented first in the statement
of financial position.
7. PAS 1 prescribes an order or format of presenting items in
the financial statements.
8. An entity may omit the notes when presenting general
purpose financial statements.
If profit or loss is P100 while other comprehensive income is
P20, total comprehensive income must be P130.
10. PAS 1 encourages, but does not require, the disclosure of an
entity's domicile and legal form, its country of incorporation
and the address of its registered office and a description of
the nature of its operations and its principal activities.

PROBLEM 2: MULTIPLE CHOICE


1. The objective of PAS 1 is
a. to ensure comparability by prescribing the basis for presentation of general purpose financial
statements.
b. to ensure the faithful representation of financial statements.
c. to ensure the relevance of information presented in the financial statements.
d. to prescribe the recognition and measurement principles
applicable to assets, liabilities, income and expenses.

2. Entity A's financial statements in the current period is comparable with Entity A's financial statements
in the previous period. This type of comparability is called
a. Inter-comparability c. Extra-comparability
b. Intra-comparability d. Intro-comparability

3. The scope of PAS 1 is


a. the preparation and presentation of general purpose financial statements.

b. the recognition, measurement and disclosure requirements for specific transactions and other
events.

c. the presentation of general purpose financial statements as well as all other information contained
in an entities annual report.

d. all of these

4. The statement of financial position is also called


a. balance sheet. c. positions statement.
b. income statement. d. all of these

5. When preparing financial statements, PAS 1 requires management to assess the entity's ability to
continue as a going concern. The assessment covers a minimum period of

a. at least one year from the end of the reporting period

b. at least two years from the end of the reporting period

c. at least five years from the end of the reporting period.


d. there is no such requirement.

6. Which of the following is not considered an appropriate application of offsetting under PAS 1?

a. Presenting a gain from the sale of a noncurrent asset net of


the related selling expenses.
b. Deducting foreign exchange losses from foreign exchange gains and presenting only the net
amount.

c. Deducting unrealized losses from unrealized gains from trading securities and presenting only the
net amount.

d. Deducting accumulated depreciation from the equipment account and presenting only the carrying
amount.

7. PAS 1 requires an entity to provide an additional balance sheet dated as of the beginning of the
preceding period if certain instances occur. Which of the following is not one of those instances?
(Assume all of the following has a material effect)
a. Retrospective application of an accounting policy.
b. Retrospective restatement

c. Reclassification of items in the financial statements

d. Change in the frequency of reporting

8. The PFRSs apply to which of the following?


a.
A management's review of the entity's financial performance during the period vis-å-vis its targets for
that period contained in the entity's annual report, which also includes the entity's financial
statements.

b.
Schedules, reconciliations and returns required by the Bureau of Internal Revenue (BIR) to
be filed together with the financial statements.

c• Environmental reports required by the Department of Environment and Natural Resources (DENR)
that are included in the entity's annual report.

d. Explanatory material and other information that


are disclosed in the notes to the financial statements.

9. This is the most commonly used method of presenting a statement of financial position. It facilitates the
computation of liquidity and solvency ratios

a. Classified presentation. c. Classified as to liquidity

b. Unclassified presentation d. Based on liquidity

10. Which of the following best reflects the definition of normal


operating cycle under PAS 1?
a. For a manufacturing entity, this is the usual time it takes for the entity -to acquire raw materials,
process those raw materials into finished goods, and sell the finished goods.
b. For a manufacturing entity, this is the usual time it takes for the entity to acquire raw materials,
process those raw materials into finished goods, sell the finished goods on account, and collect
the receivables.

c. For a manufacturing entity, this is the usual time it takes for the entity to acquire raw materials
on account and, settle the account.
d. For a manufacturing entity, this is the usual time it takes for the entity to sell finished goods
on account and collect the receivables.

PROBLEM 3: MULTIPLE CHOICE


1. Who is responsible for the preparation and fair presentation of an entity's financial statements in
accordance with the PFRSS? a. Accountant

c. Auditor
b. Management d. Government regulatory body

2. The statement of financial position may be presented either


showing current/non-current distinction (classified) or based on liquidity (unclassified). PAS 1 encourages a (an)
a. classified presentation c. combination of a and b

b. unclassified presentation. d. none of these

3. Which of the following is a current asset?


a. Deferred tax asset expected to reverse within 3 months from the reporting date

b. Property, plant and equipment


c. Non-trade note receivable due in 13 months
d. Accounts receivable

4. Which of the following statements is incorrect regarding the provisions of PAS 1?

a. An entity is required to present separate sections of profit or loss and other comprehensive income.
b. Presenting an income statement or statement of profit or loss in addition to a statement of other
comprehensive income is permitted when an entity elects to use the "two statement" presentation.

c. Presenting an income statement or statement of profit or loss alone without a statement of other
comprehensive income is allowed.
d. Presenting comprehensive income as a note disclosure only is prohibited.

5. When a separate statement of profit or loss (income statement) is presented,

a. it shall be displayed immediately before the statement presenting comprehensive income.

b. it shall be displayed immediately after the statement presenting comprehensive income.


c• it shall be displayed alone. The entity may opt not to present information on comprehensive income.
d. Any of these.

6. Which of the following is not correct when an entity opts to use the "two-statement presentation"
of income and expenses?
a. The separate income statement forms part of a complete set of financial statements and shall be
displayed immediately before the statement presenting comprehensive income.

b. The profit or loss section is not presented anymore in the statement presenting comprehensive
income.
c. The profit or loss section is required to be presented in the statement presenting
comprehensive income.
d. The separate statement presenting comprehensive income begins with the amount of profit or loss.

7. Entity A reclassifies a gain that was previously recognized in other comprehensive income to the
current period's profit or loss. According to PAS 1, how should Entity A present the reclassification
adjustment in the other comprehensive income section of the statement of comprehensive income?
a. as an addition c. only at net of tax.
b. as a deduction d. none of these

8. Which of the following is a current liability?


a. Deferred tax liability
b. An obligation for which the entity has an unconditional right to
defer.
c. A long-term obligation that becomes payable on demand
because of a breach of loan agreement but the lender agrees
before the balance sheet date to provide a grace period for the
lender to rectify the breach.
d. An obligation for which the entity has a conditional right to defer.

9. According to PAS I, items of other comprehensive income are presented according to the following
groupings

a. ordinary and extraordinary items


b. by nature and by function

c. those that are subsequently reclassified to profit or loss and those that are not
d. continuing and discontinued operations

10. When an entity changes the end of its reporting period and presents financial statements for a period
longer or shorter than one year, an entity shall disclose all of the following,

except
a. the period covered by the financial statements.

b. the reason for using a longer or shorter period.


c. the fact that amounts presented in the financial statements are
not entirely comparable.
d. a quantification of the possible adjustments that would
eliminate the effects of the longer or shorter reporting
period.

PROBLEM 4: FOR CLASSROOM DISCUSSION

Scope
1. PAS 1 applies to which of the following?
a. The preparation and presentation of general purpose
financial statements.
b. The recognition and measurement of specific assets, liabilities, income and expenses.
c. The disclosure requirements for specific transactions and
other events.
d. All of these.

General features
2. In 20x3, Entity A makes a retrospective application of an
accounting policy that has a material effect on the information in
the statement of financial position as at the beginning of the
preceding period. Entity A wishes to provide comparative
information in addition to the minimum requirement of PAS l,
i.e., Entity A will be presenting its 20x3 financial statements
together with the 20x2 and 20x1 financial statements. In this
case, the additional statement of financial position
required by PAS 1 will be dated
a. as at January 1, 20x1. c. as at January 1, 20x3.
b. as at January 1, 20x2. d. for the period ended 200

3. Entity A wants to change the presentation of, and the classification of some items in, its financial statements.
Which of the following statements is incorrect?

a. Entity A can make the change if it is required by a PFRs


b. Entity A can make the change if the change is expected to result in reliable and more relevant
information to the users of its financial statements.
c. Entity A may be required to provide an additional balance sheet dated as at the beginning of the
preceding period.
d. Entity A can make the change only if it makes an irrevocable
promise not to make another change within the next five years.

4. The financial statements of Entity A shows line described as "Other


current assets," "Other noncurrent liabilities," and "Miscellaneous
expenses." Which of the following is correct?
a. Entity A considers the items included in these line items as
dissimilar and cannot be included in material classes of similar
items and are also individually immaterial to warrant separate
presentation.
b. Entity A considers the items included in these line items as individually material but with dissimilar nature
or function.
c. Entity A considers the items included in these line items as comprising a material class of similar items.
d. This manner of presenting items is unacceptable under PAS
1.

Complete set of financial statements


5. According to PAS 1, a complete set of financial statements includes which of the following?

a. Income tax return


b. Directors' reports
c. Notes
d. All of these

Additional Statement of financial position

6. PAS I requires an entity to present an additional statement of financial position as at the beginning of the
preceding period when an entity makes any of the following, except
a. the retrospective application of an accounting policy.
b. the retrospective restatement of items in the financial statements.
c. the reclassification of items in the financial statements.
d. the prospective application of a change in accounting estimate.

Statement of financial position


7. The statement of financial position of which of the following entities does not show current and noncurrent
distinctions among assets and liabilities?
a. Banks and other financial institutions
b. Mining companies
c. Trading enterprises
d. Manufacturing firms

8. The principles of PAS 1 in relation to the classification of liabilities as current or noncurrent favor the current
classification. PAS 1 provides stricter conditions for classifying liabilities as noncurrent. Which of the
following statements best reflects a valid reason for this?
a. Noncurrent liabilities are usually more material in terms
of size compared to current liabilities.

b. Most primary users are concerned more with an entities current liabilities when making economic decisions
because of the shorter duration of time before they cause an outflow of economic resources.
c. The stricter conditions for noncurrent classification address the potential misuse of classification in
order to present favorably the entity's liquidity.
d. All of these.

Statement of profit or loss and other comprehensive income


9. Which of the following is not an acceptable method of presenting income and expenses?

a. -Presenting income and expenses that affect profit or loss and those
that are components of other comprehensive income in a single
statement.
b. Presenting an income statement in addition to a statement that presents comprehensive income.
c. Presenting an income statement alone without a statement that presents comprehensive income.
d. All of these are acceptable methods of presentation.
10. This method of presenting expenses is more difficult to apply but has the potential of providing more relevant
information to users. Its downside, however, is that it involves considerable judgment and may require arbitrary
allocations.
a. Nature of expense c. Classified presentation
b. Function of expense d. Based on liquidity

Notes

11. Which of the following is not a purpose of the notes?


a. to present information about the basis of preparation of the financial statements and the specific
accounting policies
b. to disclose the information required by PFRSs that is not presented elsewhere in the financial statements

c. to provide information that is not presented elsewhere in the


financial statements but is relevant to an understanding of any
of the financial statements.
d. to rectify inappropriate accounting policies.
PROBLEMS

PROBLEM 1: TRUE OR FALSE


1. Cash flows are presented in the statement of cash flows into four activities.
2. Non-financial institutions have the option of classifying interest income
received as either investing activities or operating activities.
3. Cash flows relating to income and expenses are normally classified as
investing activities in the statement of cash flows.
4. Only transactions that have affected cash and cash equivalents are
included in the statement of cash flows. Non-cash transactions are
excluded and disclosed only.
5. According to PAS 7, the indirect method of presenting cash flows relating
to operating activities shows each major class of gross cash receipts and
gross cash payments.

PROBLEM 2: MULTIPLE CHOICE


1. Entity A had the following balances at December 31, 20xI:
Cash in bank 35,000
Cash in 90-day money market account 75,000
Treasury bill, purchased 11/1/x1, maturing 1/31/x2 350,000
Treasury bill, purchased 12/1/x1,maturing 3/31/x2 400,000

How much is the cash and cash equivalents to be reported in Entity A's December
31, 20x1 statement of financial position?

a. 110,000 c. 460,000
b. 385,000 d. 860,000

2. Which of the following cash flows is presented in the operating activities


section of a statement of cash flows?
a. cash receipts from issuing shares or other equity instruments and cash payments
to redeem them
b. cash receipts from issuing notes, loans, bonds and mortgage payable and other
short-term or long-term borrowings, and their repayments
c. cash receipts from the sale of goods, rendering of service, or other forms of income
d. cash payments by a lessee for the reduction of the outstanding liability relating to a
lease
3. In the statement of cash flows of a non-financial institution, interest expense
paid is presented under

a. operating activities. c. financing activities.


b. investing activities. d. a or c

4. Which of the following is presented in the activities section of the statement of


cash flows?
a. Purchase of a treasury bill three months before its maturity date.
b. Exchange differences from translating foreign currency denominated cash flows.

c. Acquisition of equipment through issuance of note payable.


d. Bank overdrafts that can be offset

5. Entity A acquires equipment by paying a 10% down payment and issuing a note payable
for the balance. How should Entity A report the transaction in the statement of cash flows?

Down payment Note payable


a. Investing activities None
b. Investing activities Financing activities
c.Financing activities None

d. None None

PROBLEM 3: FOR CLASSROOM DISCUSSION

Cash and Cash Equivalents

1. Entity had the following balances at December 31, 20.


Cash on hand 300,000
Cash in bank 700,000
Cash in 90-day money-market account 500,000

Treasury bill, purchased 12/1/x2, maturing 2/28/x3 1,600,000


Treasury bond, purchased 3/1/0, maturing 2/28/x3 1,000,0000

How much cash and cash equivalents is reported in Entity December 31, 20x2
statement of financial position?
a. 3,800,000 c. 2,800,000
b. 3,100,000 d. 1,500,000

Presentation
2. Which of the following is included in the investing activities section of the
statement of cash flows?
a. Acquisition and sale of investments in held for trading securities.
b. Acquisition and sale of items of property, plant and equipment that are routinely
manufactured in the entity’s ordinary course of business and are to be held for
rentals and reclassified to inventories when the assets cease to be rented and
become held for sale.
c. Acquisition and sale of short-term investments in cash equivalents.
d. Cash inflow from repayment of loan.

3. Which of the following is included in the financing activities section of the statement of
cash flows?
a. cash payments for purchases of goods and services
b. cash receipts and cash payments in the acquisition disposal of property, plant and
equipment, inv property, intangible assets and other noncurrent assets
c. loans to other parties and collections thereof (other than loans made by a financial
institution)
d. cash receipts from issuing shares or other equity instruments and cash payments to
redeem them

4. This method of presenting cash flows from (used in) operating activities involves
adjusting accrual basis profit or loss for the effects of changes in operating assets and
liabilities and effects of non-cash items.
a. Direct method c. Inverse method
b. Indirect method d. Reverse method
5. Entity A declares cash dividends in 20x1 and pays the dividends in 20x2. How should
Entity A report the dividends paid in the statement of cash flows for
20x1 20x2

a. Operating activities None


b. Financing activities None
c. None Financing activities

d. None Operating or Financing


Chapter 5 —The Statement of Cash Flows

1. The statement of cash flows is

a. another name for statement of financial position.


b. a financial statement showing revenues earned by a business, the expenses incurred in earning the revenues,
and the resulting profit or loss.
c. a financial report showing the assets, liabilities, and equity of an enterprise on a specific date.

d. a financial statement that reports the cash inflows and outflows for an accounting period.

2. When preparing a statement of cash flows using the indirect method, the amortization of trademarks should
be reported as a/ an

a. increase in cash flows from investing activities.


b. reduction in cash flows from investing activities.
c. an addition to profit to arrive at cash flows from operating activities.
d. a deduction from profit to arrive at cash flows from operating activities.

3. In a statement of cash flows using indirect method of presenting cash flow from operating activities,
depreciation is treated as an adjustment to reported profit because depreciation

a. reduces the reported profit but does not involve an outflow of cash.

b. reduces the reported profit and involves an inflow of cash.

c. is an inflow of cash for asset replacement fund.


d. usually represents a significant portion of operating expenses.

4. In preparing a statement of cash flows, which of the following transactions would be considered an investing
activity?

a. Sale of a business segment


b. Issuance of bonds payable at a discount
c. Purchase of treasury shares
d. Sale of share capital

5. Significant amounts of non-cash investing and financing MC5 activities are


a. reported in the statement of cash flows under both investing and financing activities.
b. reported in the statement of cash flows if the indirect method is used.
c. disclosed in a note or separate schedule accompanying the statement of cash flows.
d. not reported nor disclosed.

6. Financing activities are those

a. that involve the production or purchase and the sale of goods and services to customers, including
expenditures to administer the business.
b. that involve making and collecting loans or purchasing and selling plant assets and other productive assets.
c. transactions with owners or long-term creditors of the business or that involve borrowing cash on a short-
term basis.
d. that include receipts of interest payments, cash collections from customers and receipt of cash dividends
from other companies.

7. Investing activities are

a. transactions that involve making and collecting loans or that involve purchasing and
selling plant assets and other productive assets.
b. transactions with owners or long-term creditors of the business or that involve
borrowing cash on a short-term basis.

c. activities that involve the production or purchase of merchandise and the sale of goods
and services to customers, including expenditures to administer the business.
d. cash flows such as proceeds from issuance of bonds and cash outflows such as payments
of principal amounts owed.
8. In preparing a statement of cash flows, sale of treasury shares at an amount greater than cost would be
classified as a /an

a. transfer activity.
b. operating activity.
c. investing activity.
d. financing activity.

9. A company acquired a building, paying a portion of the purchase price in cash and issuing a mortgage note
payable to the seller for the balance. In a statement of cash flows, what amount is included in investing activities
for the above transaction?

a. Cash payment
b. Acquisition price
c. Mortgage amount
d. Zero
10. In a statement of cash flows, receipts from sale of property, plant, and equipment would be classified as cash
inflows from

a. liquidating activity.
b. operating activity.
c. investing activity.
d. financing activity.

11. Under the indirect method, cash flows from operating activities would be increased by

a. gain on sale of investments.


b. increase in prepaid expenses.
c. decrease in accounts payable.
d. decrease in accounts receivable.

12. Cash inflows from investing activities result from

a. decrease in liabilities.

b. increase in liabilities.
c. decreases in non-cash assets.
d. increases in non-cash assets.

13. In a statement of cash flows, proceeds from the sale of a

a. leveraging activity.

b. operating activity.

c. investing activity.

d. financing activity.

14. A company's wages payable decreased from the beginning to the end of the year. In the company's
statement of cash flows in which the operating activities section is prepared under the direct method, the cash
paid for wages would be

a. salary expense plus wages payable at the beginning of the year.


b. salary expense plus the decrease in wages payable from the beginning to the end of the year.
c. salary expense less the decrease in wages payable from the beginning to the end of the year.
d. the same as salary expense.
15. Which of the following would be subtracted from profit when using the indirect method to derive net
cash flows from operating activities?

a. Increase in salaries and wages payable


b. Gain on sale of long-term investments
c. Decrease in net accounts receivable
d. Depreciation expense

16. The declaration of cash dividends by the board of directors

a. Would be reported as an operating activity in the statement of cash flows.


b. would be reported as an investing activity in the statement of cash flows.
c. Would be reported as a financing activity in the statement of cash flows.
d. is an activity that would not be reported in a statement of cash flows.

17. Which of the following would not appear in the statement of cash flows using the direct method?

a. Cash payments for operating expenses


b. Cash receipts from customers
c. Depreciation expense
d. Cash receipt for money borrowed from a bank
18. All of the following are cash equivalents, except

a. money market instruments.


b. commercial paper.
c. investments in BSP treasury bills.
d. investments in non-trading equity securities.

19. Would the following be added to or deducted from profit when reporting cash flow from operating activities
using the indirect method?
Depreciation expense increase in accounts receivable
a. Added Deducted
b. Added Added
c. Deducted Added
d. Deducted Deducted

20. All of the following would be reported in the financing activities section of the statement of cash flows, except

a. sale of trading securities.

b. paying a cash dividend.


c. issuing ordinary shares
d. purchasing treasury shares.
21. The following information is available from Champaca Company's accounting records for the year ended
December 31, 2014
Cash received from customers 870,000

Cash received for rent 10,000


Cash paid to suppliers and employees 510,000
Taxes paid 110,000
Gain on sale of equipment 30,000

What is the net cash flow from operating activities for 2014?
a. 220,000
b. 230,000
c. 250,000
d. 260,000

22. The following was taken from the 2014 financial statements of Tulip Company.

Accounts receivable: January 1- P216,000; December 31 P304,OOO


Cash sales and credit sales — 4,380,000
Bad debts expense — P 10,000 (No accounts receivable were written off or recovered during the year.)

What is the amount of cash collected from customers during 2014?

a. 4,478,000
b. 4,468,000
c. 4,292,000
d. 4,282,000

23. Dahlia Corporation's transactions for the year ended December 31, 2014 included the following:

• Purchased real estate for P550,000 cash which was borrowed from a bank.
• Sold investment securities for P500,000.
• Paid dividends of P600,000.
• Issued 500 ordinary shares for P250,000 cash.
• Purchased machinery and equipment for P 125,000 cash.
• Paid P450,000 toward a bank loan.
Dahlia's net cash used in investing activities for 2014 was
a. 675,000
b. 375,000
c. 175,000
d. 50,000
24. Dahlia's net cash used in financing activities for 2014 was
a. 50,000
b. 250,000
c. 450,000
d. 500,000

25. Gumamela Company's prepaid insurance was P500,000 at December 31, 2014 and P250,000 at December 31,
2013. Insurance expense was P200,000 for 2014 and P 150,000 for 2013.
What was the amount of cash disbursed for insurance reported in Gumamela's 2014 statement of cash flows under
the direct method?

a. P550,000
b. P450,000
c. P300,000
d. P200,000

26. Magnolia, Inc. has provided the following 2014 account balances for the preparation of the annual statement of
cash flows:

January 1 December 31

Accounts receivable P115,000 P145,000

Allowance for bad debts 4,000 5,000

Prepaid rent expense 62,000 41,000

Accounts payable 97,000 112,000

Magnolia's 2014 profit is P750,000. What is the 2014 net cash from operations?

a. 727,000
b. 743,000
c. 755,000
d. 757,000
27. Sunflower Company is preparing a statement of cash flows for MC27 the year ended December 31, 2014. It has
the following account balances:

Dec. 31, 2013 Dec. 31, 2014

Machinery 2,500,000 3,200,000


Accumulated depreciation 1,020,000 1,200,000
Loss on sale of machinery - 40,000

During 2014, Sunflower sold for P260,000, a machine that cost 400 000 and purchased several items o machinery.

What is the depreciation expense for 2014?


a. 180,000
b. 240,000
c. 280,000
d. 320,000
28. Use the same information given in MC 27. What is the cost of machinery purchased during 2014?

a. P 340,000
b. P 700,000
c. P 960,000
d. P1,100,000

29. Daffodil Corporation's statements of financial position as of December 31, 2014 and 2013 and information
relating to 2014 activities are presented below:

12/31/14 12/31/13
other 540,000 P 100,000
Cash and cash equivalents
Accounts receivable (net) 510,000 510,000
Inventory 680,000 600,000
Equity investments through

comprehensive income 320,000 330,000


Property, plant and equipment 1,600,000 1,000,000
Accumulated depreciation (450,000) (450,000)

120,000 120,000
Franchise
Accumulated amortization (30.000) (20.000)
Total assets 3,290,000 2,190,000

12/31/14 12/31/13
Accounts Payable 705,000 680,000
Accrued expenses 120,000 40,000

Notes payable to bank 325,000


Ordinary share capital, PIO par 800,000 700,000
Share premium 370,000 250,000
Retained earnings 950,000 490,000
Unrealized gain/ loss on equity
investments 20 000 30000

Total liabilities and equity


3,290,000 2,190,000

Information relating to 2014 activities:

• Total comprehensive income for the year was P690,000.


• Cash dividends of P240,000 Were declared and paid in 2014.
• Equipment costing P400,000 and having a carrying amount of P150,000 was
sold for P150,000
• The equity investments are carried at fair value. There were no acquisitions
nor disposals of equity securities during 2014.
• 10,000 ordinary shares were issued for P22 per share.
What is the net cash from operations in 2014?

a. P690,000
b. P925,000
c. P975,000
d. P985,000

30. Use the same information given in MC 29. What is the 2014 net cash from/ used in investing
activities?
a. 1,000,000

b. 895,000

c. 850,000

d. 815,000

31. Use the same information given in MC29. What is the 2014 net MC31 cash from (used in)
financing activities?

a. 305,000 b. 440,000

c. 455,000 d. 545,000

32. The property, plant and equipment and accumulated depreciation account balances for Marigold
Corporation are as follows:

Property, plant and equipment:


12/31/13 – 5,130,000; 12/31/14 – 4,700,000
Accumulated depreciation:
12/31/13 – 1,820,000; 12/31/14 – 1,700,000
Assuming all changes in the accounts resulted from the sale of an asset and from depreciation expense
of P80,000, what is the selling price of the asset if a P30,000 net gain resulted from the transaction?

a. 420,000
b. 360,000
c. 260,000
d. 230,000

33. The following information were taken from the 2014 operations of Morning Glory
Corporation:

Dividends revenue — P 149,000


Dividends receivable: January I—P 13,000; December 31-PI7,000
Salaries expense — P840,000
Salaries payable: January 1 — P32,000; December 31 — P53,000

What are the amounts of dividend revenue received and salaries expense paid, respectively,
that would be reported in the 2014 statement of cash flows?

a. 145,000 and 819,000


b. 153,000 and 861,000
c. 153,000 and 819,000
d. 149,000 and 840,000
34. The balance in the retained earnings at December 31, 2013 was 3,600,000 and at December 31,
2014 was 2,910,000 profit for 2014 was 2,500,000. A bonus issue was declared and distributed which
increased ordinary share capital by 1,000,000 and share premium by P550,000. A cash dividend was
declared and paid.

What is the amount of cash dividend declared and paid in 2014?


a. 1,240,000
b. 1,640,000
c. 2,210,000
d. 3,190,000
39. The profit or loss accounts of Orchid Corporation for the year 2014 included the following:
Revenues
Were 8,000,000; Operating expenses totaled 7,200,000 that included depreciation of P150,000 and
intangible asset amortization of P20 000. The Profit and loss also reported unrealized loss on equity
investments at fair value through profit or loss of P18,000 and gain on sale of equipment, P28,000.
How much is the net cash flow from operating activities?

a. 988,000
b. 970,000
c. 960,000
d. 810,000

RELATED PARTY DISCLOSURES


PROBLEM 1: MULTIPLE CHOICE
1. Which of the following best indicates that two parties are related for purposes of PAS 24?
a. One party is larger than the other.
b. The parties are a parent and a subsidiary.
c. One party has the ability to affect the financial and operating decisions of the other party
through control, significant influence or joint control.

d. One party is in the private sector and the other is a government regulatory body.

2. Which of the following are not related parties?


a. A parent and its subsidiary

b. Two or more subsidiaries with the same parent


c. A company and its Chief Executive Officer
d. Two co-venturers of a common joint venture business
3. Which of the following is not a required disclosure under PAS 24?

a. Relationships between parents and subsidiaries


b. Key management personnel compensation
c. Related party transactions (in the separate financial statements)
d. Related party transactions (in the consolidated financial statements)

PROBLEM 2: FOR CLASSROOM DISCUSSION


1. According to PAS 24, related party disclosures are necessary
a. because related party transactions may have resulted to assets and liabilities that were recognized
in the financial statements of the reporting entity.
b. to notify users of financial statements of the fact that related party transactions may not have been
made on arm's length basis.
c. to indicate the possibility that an entity's financial position and performance might have been affected
by the existence of such relationship.
d. in order to eliminate or minimize the effects of related party transactions on the financial statements
of the reporting entity.

2. What is overriding consideration when determining the existence of a related party


relationship?
a. The ability of one party to affect decisions of another party regarding relevant activities through
the existence of control, joint control or significant influence.
b. The presence of relationship either by consanguinity or affinity.

c. The presence of a significant interest by one party over the other.

d. The presence of significant business transactions and economic dependence between the parties.

3. Mr. Y and Ms. Z share joint control over Ventures, Inc. Which of the following are related
parties?

a. Mr. Y and Ms. Z c. Ventures, Inc. and PAS 24


b. Ventures, Inc. and Mr. Y d. none of these

4. Entity A is the parent company of Entity B. Which of the following is required to be disclosed
in the group's (Entity A and B's) consolidated financial statements?
a. the related party relationship between Entity A and Entity

b. the related party transactions during the period


c. the outstanding balances in (c)
d. all of these
INTERIM FINANCIAL REPORTING

PROBLEM 1: MULTIPLE CHOICE

1. Entity A wants to publish quarterly interim financial reports. Which of the


following standards may Entity A apply in preparing and presenting its interim
financial reports?

a. PAS 1 c. PFRS 1

b. PAS 34 d. a or b

2. If an entity does not prepare interim financial reports,

a. its annual financial statements would not conform to the


PFRSs.

b. its annual financial statements should not be described to have been prepared
in accordance with PFRSs

c. the conformance of its annual financial statements with the PFRSs is not
affected.

d. a and b
PROBLEM 2: FOR CLASSROOM DISCUSSION
1. Which of the following is correct regarding the provisions of PAS 34?

a. All entities should publish quarterly interim reports.


b. All publicly-listed entities should publish quarterly interim reports.
c. All publicly-listed entities should publish semi-annual interim reports.
d. PAS 34 does not require any entity to publish interim reports, and how often.

2. Interim financial reports prepared in accordance with PAS 34 shall, at a minimum, include

a. semi-annual interim financial statements.

b. complete set of financial statements.

c. condensed set of financial statements.

d. a statement of financial position and an income statement.

3. Entity A publishes quarterly interim financial reports. Entity A's annual depreciation for items of PPE
is P120,000. At the end of the first quarter, Entity A's inventories have a cost of P600,000 and a net
realizable value of P500,000. Entity A expects that the total employee bonuses (13th month pay) that
will be paid at year-end will amount to P60,000. How much is the total amount of expense to be
recognized from the items described above in Entity A's first quarter statement of profit or loss?
a. 120,000 c. 30,000
b. 135,000 d. 270,000
OPERATING SEGMENTS
PROBLEM 1:
MULTIPLE CHOICE
1. PFRS 8 requires which of the following approaches in identifying operating segments?
a. manager's approach c. direct approach
b. gentle approach d. management approach

2. According to PFRS 8, a reportable operating segment is one which


a. management uses in making decisions about operating matters.
b. results from aggregation of two or more segments and qualify under any of the quantitative thresholds
c. a and b
d. none of these

3. Which of the following is not among the quantitative thresholds under PFRS 8?

a. At least 10% of total revenues (external and internal).

b. At least 10% of the higher of total profits of segments reporting profits and total losses of segments reporting
losses, in absolute amount.

c. At least 10% of total assets (inclusive of intersegment receivables).


d. At least 10% of total revenues (external only).

4. ABC Co. operates in six geographical areas offering three major types of products and services.
Internal reports are structured based on the major types of products and services. How should ABC
Co. identify its reportable segments for external reporting in accordance with PFRS 8?

a. Based on the six geographical areas.

b. Based on the three products and services.


c. Based on quantitative threshold.

d. Based on the operating results of either the geographical areas or the products and services.

5. According to PFRS 8, disclosures for major customer shall be provided if revenues from transactions with a
single external customer amount to

a. at least 75% of the entity's external and internal revenues•

b. at least 75% of the entity's external revenues.

c. 10% or more of the entity's external revenues.


d. less than 10% of the entity's external revenues.

PROBLEM 2: FOR CLASSROOM DISCUSSION


Management approach
1. Catalyst Co. is engaged in business process outsourcing. Catalyst subcategorizes its main services into
four: Information Technology, After-sales Support, Accounting, and Offsite Data Management. Catalyst
operates in five major geographical areas: Southeast Asia, North America, South America, Australia
and Europe. Internal reports are based on these five geographical areas. What is the most appropriate basis
of segment reporting for Catalyst?
a. On the basis of the main services provided.
b. On the basis of the geographical areas of operations.
c. On the basis of the domicile country of Catalyst and the rest of the world.
d. Any of these.

Quantitative thresholds
2. Segment A qualifies under the 10% test of total revenues but not on the profit or loss and total
assets tests. Segment A
a. is not a reportable segment
b. is nonetheless included in the “all others” segment
c. may be reported as a separate segment
d. all of these

3. Information on an entity’s operating segments is shown below:


Operating segments Total Revenue Profit Identifiable assets
A 1,000,000 200,000 4,000,000
B 500,000 120,000 1,000,000
C 300,000 30,000 800,000
D 500,000 50,000 1,700,000
E 200,000 60,000 800,000
F 900,000 400,000 1,000,000
TOTALS 3,400,000 860,000 9,300,000

The reportable segments are


a. A, B and F
b. A, B, D and F
c. A, B, C, D and F
d. All segments
ACCOUNTING POLICIES, CHANGES IN ESTIMATES AND ERRORS

PROBLEM 1: MULTIPLE CHOICE


1. A change in measurement basis is most likely
a. change in accounting policy. C. error
b. change in accounting estimate. d. any of these

2. A correction of prior period error is accounted for by


a. retrospective application. c. prospective application.
b. retrospective restatement. d. impracticable application.

3. Which of the following is a change in accounting estimate?


a. Change from the cost model to the fair value model of measuring investment property.

b. Change in business model for classifying financial assets resulting to the reclassification of a
financial asset from being measured at amortized cost to fair value.
c. Change in the method of recognizing revenue from long term construction contracts.
d. Change in the depreciation method, useful life or residual value of an item of property, plant
and equipment.

4. These result from new information or new developments.


a. Changes in accounting estimates
b. Changes in accounting policies
c. Correction of errors
d. All of these

5. The effect of which of the following is presented in profit or loss in the current period (or current and
future periods/ if both are affected) rather than as an adjustment to the op balance of retained earnings.

a. Correction of a prior period error.


b. Change in accounting policy.
c. Change in accounting estimate.
d. All of these.

PROBLEM 2: FOR CLASSROOM DISCUSSION


1. According to PAS 8, in the absence of a PFRS that specifically deals with a transaction, management shall
a. refer to the concepts under the Conceptual Framework,
b. adopt the provisions of the US GAAP.

c. use its judgment in developing and applying an accounting policy that results in information that is
relevant and reliable.
d. consider the applicability of relevant accounting literature.

2. According to PAS 8, a change in accounting policy is accounted for


a. using a transitional provision, if any.
b. retrospectively.
c. prospectively, if retrospective application is impracticable.
d. a, b or c, whichever is most appropriate

3. This refers to applying a new accounting policy to transactions, other events and conditions as if that
policy had always been applied.
a. Retrospective application c. Prospective application
b. Retrospective restatement d. Impracticable application

4. According to PAS 8, a change in accounting estimate is accounted for


a. using a transitional provision, if any.
b. retrospectively.
c. prospectively.
d. a, b or c, whichever is most appropriate

5. Entity A changes its inventory cost formula from FIFO to Weighted average. How should Entity A
account for this change?
a. by retrospective restatement, as a change in accounting policy

b. by prospective application, as a change in accounting estimate


c. by retrospective application as a change in accounting policy
d. as a correction of prior period error
Interim Reporting

MCI Conceptually, interim financial statements can be described as emphasizing

a. timeliness over reliability.


b. reliability over relevance.
c. relevance over comparability.
d. comparability over verifiability.

MC2 Interim period is a financial reporting period for

a. period of any length of time.


b. a period shorter than one year.
c. a period longer than one year.
d. six months.
MC3 You are given two statements:

1. Non-compliance with IAS 34 Interim Financial Reporting indicates that the entity does not comply with the
requirements of IAS 1 Presentation of Financial Statements.
Il. IAS 34 Interim Financial Reporting requires entities whose equity or debt securities are traded in a public
capital market to publish interim reports at least as of the end of the first half of their financial year.

Indicate whether each statement is true or false.

a.Both statements are true.


b. Both statements are false.
c.Statement I is true; statement Il is false.
d. Statement I is false; statement Il is true.

MC4 Rental costs may be accrued or deferred to provide an appropriate expense in each period for

Interim Financial Reporting Year-end financial reporting

a. Yes No
b. Yes Yes
c. No No
d. No Yes
MC5 For external reporting purposes, it is appropriate to use estimated gross profit rates to determine the
cost of goods sold for
Interim financial reporting Year-end financial reporting
a. Yes Yes
b. Yes No
c. No Yes
d. No No

MC6 Sweetie adopts the calendar year as its reporting period and published interim financial report for the
quarter ended September 30, 2014. Which of the following financial reports does not bear a correct date /
period?

a. Statement of financial position as of September 30, 2014 and as of December 31,


2013.
b. Statement of financial position as of September 30, 2014 and as of September 30,
2013.
c. Statement of comprehensive income for the quarter ended September 30, 2014
and for the quarter ended September 30, 2013.
d. Statement of comprehensive income for the nine months ended September 30,
2014 and for the nine months ended September 30, 2013.

MC7 Interim financial reports should include as a minimum


a. a complete set of financial statements complying with IAS

b. a condensed set of financial statements and selected notes.


c. a statement of financial position and statement of comprehensive income only.
d. a statement of financial position, a statement of comprehensive income, and a statement of cash flows.

MC8 IAS 34 presumes that anyone reading interim financial reports

a. understand all Philippine Financial Reporting Standards.


b. have access to the records of the entity.
c. have access to the most recent annual financial statements.
d. not make decisions based on report.
MC9 Revenues received seasonally, cyclically, or occasionally within a financial year

a. shall be anticipated as of an interim date and the revenue should be recognized


equally over the interim periods.
b. shall be deferred and recognized as revenue during the final interim period of the
financial year.
c. shall be disclosed, but revenue shall be recognized when earned applying the
accounting policies as in annual.
d. shall not be disclosed, and revenue shall be recognized when earned applying the
accounting policies as in annual.
MCI0 For interim financial reporting, a loss from flash flood occurring in the third quarter should be

a. recognized in the third quarter.


b. disclosed by a note only in the third quarter.
c. recognized equally during the third and fourth quarters.
d. deferred and recognized during the last quarter of the year.

MC 11 XYZ changed its inventory costing method from FIFO to weighted average during the second quarter
of the year. For interim reporting purposes, this change shall be reflected

a. at the beginning of the second quarter of the current financial year.


b. at the beginning of the third quarter of the current financial year.
c. at the beginning of the first quarter of the current financial year.
d. at the beginning of the first quarter in the earliest comparative prior period presented by restating the
financial statements of prior interim periods and the prior interim periods of the current financial year.

MC 12 In January 2014, Victor, Inc. paid property taxes on its factory MC12 building In Jan for the
calendar year 2014 in the amount of P240,000. In the first week of April 2014, Victor made advertising
campaign and paid P600,000. These advertisements are expected to benefit operations for the remainder
of the calendar year.

How should these expenses be reflected in Victor's quarterly interim financial reports?

1 st Quarter 2nd Quarter 3rd Quarter 4th Quarter

a. P 60,000 P260,000 P260,000 P260,000

b. P 60,000 P660,000 P 60,000 P 60,000


c. P240,000 P600,000 P O 0
d. P210,000 P210,000 P210,OOO P210,000

MC13 On March 31, 2014, the end of the first quarter, Christian, Inc. estimated that its yearend bonus to
executives would be P300,000 for 2014. The actual obligation paid for the year-end bonus for 2013 was
P250,000. The estimate for 2014 is subject to yearend adjustment. What amount, if any, of expense should
be reflected in Christian's quarterly statement of comprehensive income for the three months ended March
31, 2014?

a. 0
b. 62,500
c. 75,000
d. 300,000
MC14 In 2014, Old Spice Company started operations with an inventory costing P200,OOO. For the first
quarter of 2014, the purchases and sales were as follows:

January February. March Total


Purchases P 50,000 P 380,000 P 704,000 P1, 134,000
Sales
240,000 510,000 755,000 1,505,000

The goods are sold at a gross profit of 20% of sales.


For interim statement purposes, the inventory at the end of January, February, and March, respectively are
a. P58,000; P30,000; P130,000
b. P70,000; P94,000, P0
b. P187,500; P70,000; P207,750
c. P192,000; P408,000; P604,000

MC15 New Company has calculated that total depreciation expense for the year ending December 31, 2014
will amount to P600,000, and that 2014 year-end bonuses to employees will total P 1,500,000. In New
Company's interim statement of comprehensive income for the six months ended June 30, 2014, what is the
total amount of expense relating to these two items that should be reported?

a. 0
b. P 300,000
c. 1,050,000
d. 2,100,000

MC 16 During the second quarter of 2014, Square Company sold a piece of equipment at a loss of P60,000.
What portion of the loss should Square report on its statement of comprehensive income for the second
quarter of P2014?

a. P 60,000
b. P 40,000
c. P 20,000
d. P o

MC 17 X Corporation incurs costs unevenly throughout the financ ial year. Advertising costs of P2 million were
incurred on February 28, 2014 and staff bonuses are paid at year-end based on sales. Staff bonuses are
expected to be around P30 million for the year, based on sales of P300 million. Total sales for the quarter
ending March 31, 2014 were P70 million. What costs should be included for the quarter ended March 31,
2014?

Advertising Costs Staff Bonuses


a. 2.0 m 7.5m
b. 0.5 m 7.5 m
c. 0.5 m 7.0 m
d. 2.0 m 7.0 m

MC18 An entity prepares quarterly interim financial reports in accordance with IAS 34. quarterly The entity sells
goods financial that are reports subject to warranty. The company made a provision for warranty in the first
quarter of the year 2014 at 5% of sales, as the company in the past experienced a 5% claim on warranty based
on sales. However, in the second quarter, a modification in the design of the product resulted to a design fault
and the company expected the warranty claims to increase to 10% for the whole year 2014. Sales in the first and
second quarters were PI0 million and P 15 million, respectively. What would be the warranty expense charged
in the second quarter's interim financial statements?

a. P2.0 million
b. P 1.5 million
c. PI .25 million
d. P0.75 million

MC20 An entity's accounting year ends on December 31, and it is currently preparing interim financial
statements for the half-year to June 30, 2014. The price of its product tends to vary. At June 30, 2014, it has
inventories of 100,000 units, at a cost per unit of P 14. The net realizable at June 30, 2014 is P 12 per unit. The
expected net realizable value of these inventories at December 31, 2014 is P 15.50 per unit.

At what amount should these inventories be presented in the interim statement of financial position at June 30,
2014?

a. 1,200,000
b. 1,375,000
c. 1,400,000
d. 1,550,000

Operating Segments
MC21 Under IFRS 8, Operating Segments, which among the following is not necessarily a characteristic of
an operating segment?

a. A major portion of its revenue is derived from sales to outside customers.


b. It is engaged in business activities from which it may earn revenues and incur
expenses.
It is one whose operating results are reviewed by the entity's chief operating decision maker.
d. It is a component of an entity for which discrete financial information is available.

MC22 Which of the following is not a valid statement regarding reporting by operating segments?

a. Operating segments are identified based on the management structure for


internal reporting purposes.
b. Some components of an entity may be grouped together to form one operating
segment.
c. Reporting by operating segments is encouraged for enterprises whose equity
or debt securities are publicly traded.
d. A segment that is not identified as reportable may be combined with other
similar segments or may be presented as part of unallocated reconciling item.

MC23 Segment result is described as

a. total segment income.


b.segment revenue less segment expenses.
c. segment profit after any adjustments for non-controlling interest.
d.segment profit after any adjustments for income tax.
MC24 Under IFRS 8, separate segments of an entity must be identified as reportable segments until at least

a. 100% of total entity revenue is included.


b. 80% of total entity assets are included.
c. 75% of total external revenue is included.
d. 70% of total entity assets are included.
entity is engaged in the manufacturing industry and has An
MC25 recently purchased an 80% holding in a retail sales group. This does not meet any of the
quantitative criteria for a group reportable segment. Can the entity disclose the retail sales group
as a separate operating segment?

a. No, because it does not meet any of the quantitative


criteria.
b. Yes, if management believes that the retail sales group is a distinguishable component
of the entity.
c. No, because the entity holds only 80% equity in the retails sales group.
d. No, because the retail sales group is not engaged in the manufacturing industry.
MC26 Under IFRS 8 Operating Segments, a segment is reportable if

a. total revenue is 10% or more of total revenue, internal and external, of all segments.
b. total revenue is 10% or more of the combined external revenue of all segments.
c. liabilities are 10% or more of total liabilities.
d. assets are 5% of more of total assets.

MC 27 An operating segment is a reportable segment if

a. its operating profit is 10% or more of the combined operating profit of all segments
that report a profit.
b. its operating loss is 10% or more of the combined operating losses of segments that
incurred an operating loss.
c. the absolute amount of its operating profit or loss is 10% or more of the company's
combined operating profit or loss
d. the absolute amount of its profit or loss is 10% or more of combined profits of all
segments operating at a profit or combined losses of all segments operating at a loss,
whichever is higher in absolute amount.

MC28 The following information pertains to Blue Company and its divisions for the year ended December
31, 2014:
Sales to unaffiliated customers 1,000,000
Intersegment sales of products similar to those sold to unaffiliated customers 300,000

Interest revenue (reviewed by chief operating officer) 20,000

Blue and all of its divisions are engaged solely in manufacturing operations.

Blue is a reportable segment if that segment's revenue exceeds

a. P100,00
b. P102,000

c. P130,000
d. P132,000

MC29 Green Company reports operating profit as to industry segments in its supplementary financial
information annually. The following information is available for 2014:
Sales Traceable Costs
Segment 1 P 750,000 P450,000
Segment 2 500,000 225,000
Segment 3 250 000 125 000
1,500,000 800,000
Additional expenses not included above are as follows:
Indirect operating expenses P240,000
General corporate expenses 180,000
Interest expense 96,000

Green allocates common costs based on the ratio of a segment’s sales to total sales.
What should be the operating profit for Segment 2 for 2014?

a. P103,000
b. P135,000
c. P163,000
d. P195,000
MC30 which purple is Company appropriately operates regarded in six different as an operating industries,
each of which is appropriately regarded as an operating segment. Purple's 2014 combined sales for all
segments aggregated PIO million. Segment No. 4 had sales of P2.0 million and traceable costs of P900,000.
Combined common costs for all segments totaled P3.O million. Common costs are allocated among the six
segments on the basis of each segment's percentage of Purple's total sales, which is an acceptable
allocation method.
How much should be reported as Segment No. 4's operating income for 2014?

a. 500,000
b. 1,100,000
c. 1,220,000
d. 1,400,000
MC31 Yellow Company has five business divisions. The following data with regard to its operating
segments (all of which are engaged in manufacturing) for the year ended December 31, 2014 are as
follows:

DIVISION SALES COST AND EXPENSES INTEREST


EXPENSE

1
2
3
4
5
Total

Yellow Company does not adopt the practice of reporting interest expense to the chief operating
officer of the enterprise. Yellow Company shall consider a segment reportable if its operating profit
is at least

a. 2,500,000
b.1,900,000
c.1,300,000
d. P 570,000

MC33 Billy Company, a corporation listed in the stock exchange, reports operating results from its
Product X Industry activities to its chief operating decision maker. The segment information for the
year and the information for the enterprise follow:

Product X Industry Total for the Entity


Revenue 3,500,000 40,000,000

Profit (all segments reported profit) 900,000 9,600,000

Assets 1,800,000 17,500,000

Number of employees 2,500 20,000

Which piece of information determines for Billy Company that the Product X Industry activities are a
reportable segment?

a. Revenue
b. Profit
c. Assets
d. Number of employees

Chapter 6 - Cash and Accrual Accounting/Single-entry


MCI Under the accrual basis, revenues are recognized when they are

a. collected.
b. earned.
c. earned and collected.
d. earned and become measurable.

MC2 Under the cash basis, revenues are recognized when they are

a. collected.
b. earned.
c. earned and collected.
d. earned and become measurable.

MC3 The recognition of expenses, under accrual accounting, is based on three principles: direct
matching, systematic and rational allocation and immediate recognition. The direct matching
principle requires that expenses be recognized

a. when they are paid by the enterprise.


b. in the same period that the costs expire or assets are used.
c. in the same period in which the revenues are recognized that the expenses help to
produce.
d. in the same period that the revenue is received that the expenses help to produce.

MC4 Under the cash basis, expenses are recognized when

a. they are paid by the entity.


b. the costs expire or assets are used.
c. the revenues are recognized that the expenses help to produce.
d. cash is received from revenues that the expenses help to produce.

MC5 Accrual basis profit is most useful for

a. determining the amount of income tax payable to the government


b. predicting the performance of an entity for the succeeding reporting period.
c. determining the amount that will paid as interest to creditors and dividends to stockholders
d. predicting the long-term performance of an enterprise.
MC6. The net worth method, otherwise known as the capital maintenance approach, is a concept which
a. profit equals the change in the market value of the net assets during a period.
b. profit is measured as the amount that an enterprise could distribute to its owners and be as well off at the end
of the period as it was at the beginning of the period.
c. the financial statement effects of business events classified as revenues, gains, expenses and losses, which are
used to measure and define profit
d. market values adjusted for the effects of inflation or deflation are used to measure profit

MC7 A company's merchandise inventory increased during the period, while its accounts payable
decreased during the period. How would these increases or decreases be added to or deducted from cash
payment to merchandise suppliers to arrive at accrual basis of cost of goods sold?

Increase in inventory Decrease in accounts payable

a. Added Deducted

b. Added Added
c. Deducted Deducted
d. Deducted Added

MC8 When converting from cash basis to accrual basis of accounting which of the following
adjustments should be made to cash collections from customers to arrive at the accrual
basis sales?

a. Add ending accounts receivable


b. Subtract ending accounts receivable
c. Add beginning accounts receivable
d. Deduct beginning balance of advances from customers

MC9 When converting from cash basis to accrual basis of accounting, which of the following
adjustments should be made to cash payments for expenses to arrive at the accrual basis
expenses?

a. Add beginning prepaid expenses


b. Subtract beginning prepaid expenses

c. Add beginning accrued expenses


d. Add ending prepaid expenses
MC 10 Over the life of the enterprise,

a. total profit using the accrual basis would be more than total profit using the cash
basis.

b. total profit using accrual basis would be less than the total profit using the cash basis.

c. total profit using the accrual basis would equal the total profit using the cash basis.
d. total profit using the accrual basis would either be more than or less than the total profit using
the cash basis.

MC11 Ensure Company maintains a medical and dental clinic and keeps limited accounting records. Its
assets and liabilities at the beginning and end of the current year are as follows:
Beginning End
Cash in bank P12,OOO P (5,000)
Accounts receivable 68,000 70,000
Medical supplies 30,000 15,000
Accounts payable 40,000 20,000
Notes payable - bank 20,000 25,000
Medical equipment (net) 150,000 125,000
During the year, the owner withdrew cash of P 12,000 and made additional investment of P50,000. The
profit (loss) of Ensure Company for the year is
a. 8,000
b. (2,000)
c. (68,000)
d. (78,000)

MC 12 The following information pertains to Birch Tree Company's 2014 sales:

Cash Sales
Gross 80,000

Returns and Allowances 4,000

Credit Sales

Gross 120,000

Discounts 6,000
On January 1, 2014, customers owed Birch Tree P40,000. On December 31, 2014, customers owed Birch
Tree P30,000. Birch Tree uses the direct write-off method for bad debts. No bad debts were recorded in
2014.

Under the cash basis, the net revenue to be reported for 2014 is

a. P200,000
b. P190,000
c. PI 70,000
d. P 76,000

MC13 Bear Brand, Inc. owns an office building and leases the offices under a one-year rental agreement. Not all
tenants make timely payments of their rent. Bear Brand's statement of financial position contained
the following data:
2014

P 96,000 P124,000
Rentals Receivable 320,000 240,000
Unearned Rentals
During 2014, Bear Brand received P800,000 cash from tenants.
The amount of Rental Revenue for 2014 is

a. P692,000
b. P748,000
c. P852,000
d. P908,000

MC 14 The accrual profit or loss of Carnation, Inc. included the following expenses for 2014:
Depreciation expense - P65,000; Salaries and wages - P 189,000; Interest expense - P36,000.

The comparative statements of financial position reported the following related accounts:
12/31/14 12/31/13
Accumulated depreciation P96,000 P154,000

Accrued salaries and wages 8,000 12,000

Accrued interest payable 10,500 7,000

Considering only the data above, a pure cash basis income statement would report expenses for
2014 of

a. P224,500
b. P225,500
c. P231,500
d. P243,500

MC15 The following data relate to store equipment of Progress Company for 2014:
2013 2014

Store equipment — cost P145,000 P175,000


Accumulated depreciation 58,000 60,000

Store equipment costing P30,000 was sold for P21,000 resulting in a gain of P3,000.

The depreciation expense on store equipment for 2014 is

a. P14,000
b. PI 1,000
c. P10,000
d. P 8,000
MC16 Milkmaid, Inc. maintains its accounting records on the cash basis but restates its financial statements
to the accrual method of accounting. Milkmaid had P600,000 cash basis pretax income for 2014.

The following information pertains to Milkmaid for the years ended December 31, 2014 and 2013:

2014 2013
Accounts receivable P400,000 P200,000
Accounts payable 150,000 300,000
Under the accrual method, Milkmaid should report in its
December 31, 2014 profit or loss a pretax profit of

a. P250,000
b. P550,000
c. P650,000
d. P950,000

MCI 7 The current year's operations of Cerelac, Inc.:


December 31 January 1
Accounts receivable P 51,000 P 45,000
Accounts payable 60,000 33,000
Accrued wages - 7,000
Collections from customers 794,000
Total cash payments 715,000
Selling expenses of PI 44,000 are 45% of gross profit. Administrative expenses are 15% of sales. This
amount includes depreciation which is 20% of administrative expenses. There are no unpaid selling and
administrative expenses as of December 31. Inventory per physical count at year-end totaled P125,OOO.

Total sales for the period is

a. 705,000 b. 725,000
c. 788,000 d. 800,000
MC18 Use the same information given in MC17. Total purchases for the period is

a. P495,000
b. p496,440
c. P499,200
d. P504,600

MCI 9 Use the same information given in MC 17. The cost of goods sold for the period is

a. P480,000
b. P468,000
c. p445,000
d. P400,000

MC20 The following changes in Alaska Company's account balances occurred during 2014:
Increase
Assets P890,000
Liabilities 270,000
Share capital 600,000
Share premium 60,000

Except for a P130,000 dividend payments and the year's earnings, there were no changes in retained
earnings for 2014.

What was Alaska's profit for 2014?

a. P 40,000
b. P 90,000
c. P130,000
d. P170,000

MC21 Following for are six data months relating which to the operations of Centrum Company for six
months which started the on July 1, 2014:

Cash receipts:
Investment by owner P250,000
Collections on sales on account 310,000
Cash sales proceeds of a note payable dated October 1, 2014 and due October 1, 85,000
2015, discounted at 18%

Cash disbursements: 24,600


Purchase of land and building on July 1, 2014 P240,000
20% down payment on furniture and fixtures
purchased on installment on July 1, 2014 4,000
On accounts payable 280,000
For other operating expenses 45,000
Investment by

Of the sales on account, P4,000 was returned because of poor quality and there was a purchase return of
P5,000. On December 31, 2014, the following balances were available:

Accounts receivable, P66,000; Accounts payable, P67,000; Accrued other operating expenses, P3,500.

The land has an allocated cost of P40,000. Annual depreciation is 4% on the building and 10% on the
furniture and fixtures.

Inventory on December 31, 2014 is P21,700.

Gross sales for the period amounted to

a. P376,000

b. P380,000
c. P461,000
d. P465,000

MC22 Use the same information given in MC21. Gross purchases for the period amounted to

a. P347,000
b. 352,000
d. P377,000
e. P382,000
MC23 Use the same information given in MC21. Cost of sales for the period is

a. P355,300
b. P350,300
c. P325,300
d. P320,300

MC24 Use the same information given in MC21. The total operating expenses for the period is

a. P46,500
b. P48,500
c. P53,500
d. P58,500

MC25 M. Nan keeps single-entry records for his business. The accounts payable per files on May 1 was PI
10,000, and on May 31 was P 140,000. During the month, P45,000 was paid for cash purchases; P22,500 was
allowed on purchase returns; P280,000 was paid on accounts payable; and P 10,000 was paid for freight-in.
The inventory on May 1 was P50,000 and on May 31 was P60,000.

What is the cost of sales for the month of May?

a. P355,000
b. P175,000
c. P345,000
d. P365,000

MC26 Neslac, Inc. owns an office building and leases the offices under a one-year rental agreement. Not all
tenants make 'timely payments of their rent. During 2014, Neslac received P800,000 cash from tenants.
The company's statement of financial position at December 31, 2013 reported rent receivable of
p96,000 and unearned rent revenue of P320,000. The statement of financial position at December 31,
2014 and the statement of comprehensive income for the year ended December 31, 2014 reported rent
receivable and rent revenue of P 124,000 and P908,000, respectively.

How much was Neslac, Inc.'s unearned rent revenue at December 31, 2014?

P112,000
b. P168,000
e. P240,000
f. P908,000

MC27 The following data are obtained from the single entry set of books kept by the proprietor of Promil
Company for 2014:

Accounts receivable of PI 20,000 were written off as uncollectible, returns of P320,OOO were
made on merchandise sales and an allowance of P80,OOO was received on merchandise
purchases.
The gross sales for 2014 amounted to

a. 5,440,000
b. 5,500,000
c. 5,580,000
d. 5,620,000

MC28 Nido Corporation provided you with the following summary of total assets and liabilities at January 1,
2014 and at December 31, 2014:

Assets Liabilities
January 1, 2014 9,000,0000 3,200,000
December 31, 2014 12,000,000 4,500,000

During 2014, Nido issued 10,000 shares of its P 100 par ordinary share at P 150 per share and declared
dividends of P280,000. There were no other changes affecting the equity accounts.

The profit for 2014 is

a. P 80,000
b. P 420,000
c. P 480,000
d. P 980,000

MC29 Marvell Company's total equity increased by P320,000 during 2014. New shareholder investment
during the year totaled P650,000. Total revenues during the year were and total expense were
4,600,000. Cash increased by P75,000 during the year.

What amount of dividends did Maxwell Company declare during 2014?

a. P330,000
b. P655,000

c. P730,000

d. P737,500
MC30 The changes in are the taken account from balances the and accounts the following additional
information are taken from the accounts of Great Taste Corporation for the year 2014:
Increase
(Decrease)
Cash
P 142,500
Accounts receivable (30,000)
Inventory 202,500
Buildings and equipment, net 630,000
Accounts payable (172,500)
Bonds payable 375,000
Share capital 300,000
Share premium 45,000
Dividends for 2014 were P82,500. There were no transactions affecting retained earnings other than
dividends and profit.

The profit for 2014 is

a. P 397,500
b. P 480,000
c. P 562,500
d. 1,410,000

MC31 During 2014, Kopiko Company, a service organization, had P200,000 in cash sales and 3,000,000 in
credit sales. The accounts receivable balances were P400,000 and P485,000 at December 31, 2013
and 2014, respectively.

If Kopiko desires to prepare a cash basis income statement, how much should be reported as sales for 2014
on a cash basis?

a. 3,285,000
b. 3,115,000
c. 3,200,000
d. 3,215,000

MC32 Under the accrual basis, rental income of Macho Company for the calendar year 2014 is P60,000.
Additional information regarding rental income follows:
Unearned rental income, January 1, 2014 P5,000
Unearned rental income, December 31, 2014 7,500
Accrued rental income, January 1, 2014 3,000
Accrued rental income, December 31, 2014 4,000
How much actual cash rental was received by Macho Company in 2014?

a. P58,500
b. P62,500
c. P61,500
d. P65,500

You might also like